Difference between revisions of "2018 AMC 10B Problems/Problem 21"

(Solution 3)
m (Solution 3)
Line 12: Line 12:
  
 
==Solution 3==
 
==Solution 3==
Again, recognize <math>323=17 \cdot 19</math>. The 4-digit number is even, so its prime factorization must then be <math>17 \cdot 19 \cdot 2 \cdot n</math>. Also, <math>1000\leq 646n \leq 9998</math>, so <math>2 \leq n \leq 15</math>. Since <math>15 \cdot 2=30</math>, the prime factorization of the number after 323 needs to have either 17 or 19. The next highest product after 17 and 19 is <math>17 \cdot 2  \cdot 10 = 17 \cdot 20=340</math> or <math>19 \cdot 2  \cdot 9 = 19 \cdot 18=342</math> <math>\implies \boxed{\text{(C) }340}</math>.  
+
Again, recognize <math>323=17 \cdot 19</math>. The 4-digit number is even, so its prime factorization must then be <math>17 \cdot 19 \cdot 2 \cdot n</math>. Also, <math>1000\leq 646n \leq 9998</math>, so <math>2 \leq n \leq 15</math>. Since <math>15 \cdot 2=30</math>, the prime factorization of the number after 323 needs to have either 17 or 19. The next highest product after 17 and 19 is <math>17 \cdot 2  \cdot 10 =340</math> or <math>19 \cdot 2  \cdot 9 =342</math> <math>\implies \boxed{\text{(C) }340}</math>.  
  
  

Revision as of 22:29, 11 November 2019

Problem

Mary chose an even $4$-digit number $n$. She wrote down all the divisors of $n$ in increasing order from left to right: $1,2,...,\dfrac{n}{2},n$. At some moment Mary wrote $323$ as a divisor of $n$. What is the smallest possible value of the next divisor written to the right of $323$?

$\textbf{(A) } 324 \qquad \textbf{(B) } 330 \qquad \textbf{(C) } 340 \qquad \textbf{(D) } 361 \qquad \textbf{(E) } 646$

Solution 1

Prime factorizing $323$ gives you $17 \cdot 19$. The desired answer needs to be a multiple of $17$ or $19$, because if it is not a multiple of $17$ or $19$, the LCM, or the least possible value for $n$, will not be more than 4 digits. Looking at the answer choices, $\fbox{(C) 340}$ is the smallest number divisible by $17$ or $19$. Checking, we can see that $n$ would be $6460$.

Solution 2

Let the next largest divisor be $k$. Suppose $\gcd(k,323)=1$. Then, as $323|n, k|n$, therefore, $323\cdot k|n.$ However, because $k>323$, $323k>323\cdot 324>9999$. Therefore, $\gcd(k,323)>1$. Note that $323=17\cdot 19$. Therefore, the smallest the gcd can be is $17$ and our answer is $323+17=\boxed{\text{(C) }340}$.

Solution 3

Again, recognize $323=17 \cdot 19$. The 4-digit number is even, so its prime factorization must then be $17 \cdot 19 \cdot 2 \cdot n$. Also, $1000\leq 646n \leq 9998$, so $2 \leq n \leq 15$. Since $15 \cdot 2=30$, the prime factorization of the number after 323 needs to have either 17 or 19. The next highest product after 17 and 19 is $17 \cdot 2  \cdot 10 =340$ or $19 \cdot 2  \cdot 9 =342$ $\implies \boxed{\text{(C) }340}$.


You can also tell by inspection that $19\cdot18 > 20\cdot17$, because $19\cdot18$ is closer to the side lengths of a square, which maximizes the product.

~bjhhar

Video

https://www.youtube.com/watch?v=KHaLXNAkDWE

See Also

2018 AMC 10B (ProblemsAnswer KeyResources)
Preceded by
Problem 20
Followed by
Problem 22
1 2 3 4 5 6 7 8 9 10 11 12 13 14 15 16 17 18 19 20 21 22 23 24 25
All AMC 10 Problems and Solutions
2018 AMC 12B (ProblemsAnswer KeyResources)
Preceded by
Problem 18
Followed by
Problem 20
1 2 3 4 5 6 7 8 9 10 11 12 13 14 15 16 17 18 19 20 21 22 23 24 25
All AMC 12 Problems and Solutions

The problems on this page are copyrighted by the Mathematical Association of America's American Mathematics Competitions. AMC logo.png